Physiology 001 - Introduction to Human Physiology » Summer 2021 » Exam 2 (B)

Need help with your exam preparation?

Question #1
Heroin causes similar, but much stronger effects to the nervous system as endogenous opioids. Heroin's mechanism of action might be that it acts as an:
A.   Any of these are possible
B.   agonist to serotonin receptors.
C.   agonist to glycine receptors.
D.   agonist to the endogenous opioid receptors.
E.   agonist to epinephrine receptors.
Question #2
An action potential in a neuronal membrane differs from a graded potential in that
A.   action potentials vary in size with the size of a stimulus, while graded potentials do not.
B.   an action potential has a threshold, whereas a graded potential is an all-or-none phenomenon.
C.   an action potential requires the opening of Ca2+ channels, whereas a graded potential does not.
D.   movement of Na+ and K+ across cell membranes mediate action potentials, while graded potentials do not involve movement of Na+ and K+.
E.   an action potential is propagated without decrement, whereas a graded potential decrements with distance.
Question #3
You are studying the role of a newly discovered neurotransmitter. You find that there are receptors for this neurotransmitter on interneurons in the brain and that when it binds, it causes the opening of K+ channels. What can we expect of the postsynaptic cells under influence of this neurostransmitter?
A.   There will be no change to the membrane potential in the postsynaptic cell.
B.   The postsynaptic cell will immediately undergo an action potential.
C.   The postsynaptic cell will undergo an EPSP.
D.   The postsynaptic cell will undergo an IPSP.
Question #4
Which is one of the major inhibitory neurotransmitters in the CNS?
A.   Gamma-aminobutyric acid (GABA)
B.   Dopamine
C.   Beta-endorphin
D.   Glutamate
E.   Norepinephrine
Question #5
What is the definition of a "motor unit"?
A.   A single muscle fiber plus all of the motor neurons that innervate it
B.   All of the motor neurons supplying a single muscle
C.   All of the muscles that affect the movement of any given joint
D.   A single motor neuron plus all the muscle fibers it innervates
E.   A pair of antagonistic muscles
Question #6
Which of the following statements is FALSE?
A.   A single smooth muscle cell may be innervated by both a sympathetic neuron and a parasympathetic neuron.
B.   Ca2+ that activates contraction of smooth muscles can come from either the ECF or from the sarcoplasmic reticulum.
C.   Contractile activity of smooth muscle cells does not normally require Ca2+.
D.   In the absence of any neural input, skeletal muscle cannot generate tension.
E.   Synaptic input onto skeletal muscle cells is always excitatory, whereas inputs to smooth muscle cells may be either excitatory or inhibitory.
Question #7
A threshold stimulus applied to an excitable membrane is one that is just sufficient to
A.   trigger an action potential.
B.   cause a change in membrane potential.
C.   be conducted to the axon hillock.
D.   trigger an excitatory postsynaptic potential.
E.   depolarize a dendrite.
Question #8
Compared to type 1 (slow oxidative) skeletal muscle fibers, how are type 2X (fast-glycolytic) skeletal muscle fibers different?
A.   Type 2X fibers have more abundant myoglobin.
B.   Type 2X fibers have more abundant mitochondria.
C.   Type 2X motor units contain fewer fibers per alpha motor neuron
D.   Type 2X fibers are smaller in diameter.
E.   Type 2X fibers fatigue more readily.
Question #9
In skeletal muscle cells, calcium initiates contraction by binding to
A.   myosin.
B.   tropomyosin.
C.   actin.
D.   the thick filament.
E.   troponin.
Question #10
Which is most directly responsible for the falling (repolarizing) phase of the action potential?
A.   Voltage-gated Na+ channels are opened.
B.   The permeability to Na+ increases greatly.
C.   ATPase destroys the energy supply that was maintaining the action potential at its peak.
D.   The permeability to K+ increases greatly while that to Na+ decreases.
E.   The Na+, K+ pump restores the ions to their original locations inside and outside of the cell.
Question #11
In a laboratory experiment you have treated a muscle cell with a calcium channel blocker. The membrane still depolarizes, but the concentration of calcium never increases in the cytosol. Which of the following describes what will happen next?
A.   Binding of myosin to actin will take place.
B.   Contraction will occur, but the muscle will be stuck in the contracted state and unable to relax.
C.   A single twitch in skeletal muscle but no sustained contraction.
D.   Only one cross-bridge cycle will occur but no second cycle. 
E.   Tropomyosin will continue to cover the myosin binding sites on actin and no cross-bridges will form.
Question #12
Which of the following statements about the refractory period of a membrane is TRUE?
A.   The refractory period prevents the action potential from spreading back over the part of the membrane that just underwent an action potential. The relative refractory period refers to the period of time during which another action potential can be initiated in that part of the membrane that has just undergone an action potential if a stronger than normal stimulus is applied. The refractory period places an upper limit on the frequency with which a nerve cell can conduct action potentials. The absolute refractory period refers to the period of time during which another action potential cannot be initiated in that part of the membrane that is undergoing an action potential, no matter how great the strength of the stimulus.
B.   The absolute refractory period refers to the period of time during which another action potential cannot be initiated in that part of the membrane that is undergoing an action potential, no matter how great the strength of the stimulus. The refractory period prevents the action potential from spreading back over the part of the membrane that just underwent an action potential.
C.   The relative refractory period refers to the period of time during which another action potential can be initiated in that part of the membrane that has just undergone an action potential if a stronger than normal stimulus is applied
D.   The refractory period places an upper limit on the frequency with which a nerve cell can conduct action potentials.
Question #13
Which is a TRUE statement about excitation-contraction coupling?
A.   Calcium-induced calcium release plays a role in cardiac muscle cells, as well as in some smooth muscle cells.
B.   In skeletal muscle cells, excitation-contraction coupling begins when an action potential propagates along the sarcoplasmic reticulum membrane.
C.   In smooth muscle cells, it must be preceded by an action potential in the cell membrane.
D.   In skeletal muscle cells, it requires the influx of extracellular calcium ion.
E.   In all kinds of muscle it requires the entry of calcium from the extracellular fluid.
Question #14
Which of these is a major difference between smooth muscle and skeletal muscle?
A.   Myosin is the main regulatory protein in skeletal muscle.
B.   Skeletal muscle usually exhibits spontaneous activity, while smooth muscle cannot contract spontaneously. 
C.   Myosin is the main regulatory protein in smooth muscle.
D.   Only skeletal muscle requires increased calcium ion concentration in the cytosol for contraction.
E.   Only skeletal muscle has both actin and myosin.
Question #15
Which best describes temporal summation?
A.   It always brings a postsynaptic cell to threshold.
B.   A synapse is stimulated a second time before the effect of a first stimulus at the synapse has terminated.
C.   It only refers to addition of EPSPs.
D.   Two synapses on different regions of a cell are stimulated at the same time.
E.   The size of an EPSP depends on the size of the stimulus.
Question #16
Which of the following statements regarding myosin in skeletal muscle is TRUE?
A.   The rate of ATP hydrolysis by myosin is the same in all types of skeletal muscle.
B.   Myosin is an ATPase.
C.   All of the myosin cross-bridge heads in a thick filament are oriented and rotate in the same direction.
D.   Myosin cross-bridge heads contain two binding sites, one for actin and one for tropomyosin.
E.   Troponin covers the binding site on myosin molecules until Ca2+ binds to troponin to remove it from its blocking position.
Question #17
What is the function of the transverse tubules in a skeletal muscle fiber?
A.   They run in parallel with the myofibrils, and have abundant Ca2+-ATPase proteins for pumping Ca2+ back into the sarcoplasmic reticulum.
B.   They store the calcium ions that are the main source of activation for the cross-bridge cycle.
C.   They manufacture and store ATP.
D.   They allow action potentials to propagate deep into the center of skeletal muscle cells.
E.   They form the Z lines that mark the end of each sarcomere.
Question #18
Which of the following statements concerning the permeability of a typical neuron membrane at rest is TRUE?
A.   The permeability to Na+ is much greater than the permeability to K+.
B.   All of the K+ channels in the membrane are open.
C.   There is equal permeability to Na+ and K+.
D.   The voltage-gated Na+ channels are in the inactivated state.
E.   Most of the voltage-gated Na+ channels are in the closed state.
Question #19
Sarah is a girl with dreams of becoming an Olympic marathon runner. She just learned that she has a rare genetic mutation and does not produce functional creatine phosphate. What portion of Sarah's marathon experience will be effected?
A.   No effects on her running
B.   All muscle contractions after Sarah reaches fatigue (about an hour into her run)
C.   Just the first few seconds of exercise
D.   All muscle contractions after Sarah reaches her maximum heart rate (about 10 minutes into her run)
E.   Every moment of Sarah's runs
Question #20
Tetanus toxin inhibits acetylcholinesterase activity. What will a person suffering from tetanus toxin poisoning look like?
A.   All muscles of the body might exhibit frequent small twitches.
B.   All muscles of the body might present a limp, relaxed state known as flaccid paralysis.
C.   All muscles of the body might present a tense, fully contracted state.
D.   All muscles of the body will be completely normal in function.
Question #21
Which is a TRUE statement about smooth muscle?
A.   The myosin in smooth muscle requires phosphorylation before it can bind to ATP.
B.   Smooth muscle is striated.
C.   Smooth muscle does not have thick and thin filaments.
D.   Changes in cytosolic calcium do not regulate cross-bridge activity in smooth muscle.
E.   Smooth muscle does not use troponin-tropomyosin to regulate cross-bridge activity.
Question #22
Which is one of the most abundant excitatory neurotransmitters in the CNS?
A.   Norepinephrine
B.   Gamma-aminobutyric acid (GABA)
C.   Glutamate
D.   Endorphin
E.   Dopamine
Question #23
Which is TRUE about typical, resting neurons?
A.   The plasma membrane is completely impermeable to sodium ions.
B.   The plasma membrane is completely impermeable to potassium ions.
C.   The plasma membrane is most permeable to sodium ions.
D.   The permeability of the plasma membrane to potassium ions is much greater than its permeability to sodium ions.
E.   The concentration of sodium ion is greater inside the cell than outside.
Question #24
Which of the following choices lists a correct sequence of events following the depolarization of transverse tubules in excitation-contraction coupling in cardiac muscle?
A.   Calcium release into cytosol, actin and myosin attach, thin filaments slide toward the middle of sarcomeres, calcium ion influx through sarcolemma
B.   Calcium release into cytosol, calcium ion influx through sarcolemma, actin and myosin attach, thin filaments slide toward the middle of sarcomeres
C.   Calcium ion influx through sarcolemma, calcium release into cytosol, actin and myosin attach, thin myofilaments slide toward the middle of sarcomeres
D.   Calcium release into cytosol, actin and myosin attach, calcium ion influx through sarcolemma, thin myofilaments slide toward the middle of sarcomeres
E.   Actin and myosin attach, thin filaments slide toward the middle of sarcomeres, calcium release into cytosol
Question #25
Myasthenia gravis is an autoimmune disease in which the immune system gradually destroys the receptors for acetylcholine at the neuromuscular junction. Which of the following drugs might initially be useful in treating the symptoms of this disease?
A.   Atropine (a muscarinic acetylcholine receptor antagonist)
B.   A drug that inhibits release of acetylcholine
C.   A nicotinic acetylcholine receptor antagonist
D.   A drug that inhibits acetylcholinesterase
E.   Curare
Question #26
If you block Ca2+ channels on an axon, which will not occur?
A.   The depolarization phase of the action potential
B.   The exocytosis of neurotransmitter
C.   All of these will not occur
D.   The repolarization phase of the action potential
E.   The graded potential
Question #27
Which occurs FIRST as a result of ATP binding to myosin?
A.   Actin dissociates from myosin.
B.   Cross-bridge heads are cocked into an "energized" state.
C.   Actin binds to myosin.
D.   Cross-bridges rotate, sliding past the thin filament.
E.   Ca2+ is released from the sarcoplasmic reticulum.
Question #28
Inside a neuron, calcium binding sites are likely found on
A.   lysosomal proteins.
B.   kinesin proteins.
C.   dyenin proteins.
D.   smooth endoplasmic reticulum.
E.   proteins associated with synaptic vesicles.
Question #29
During the rising (depolarizing) phase of a neuronal action potential,
A.   PK+ becomes much greater than PNa+.
B.   K+ flows rapidly into the cell.
C.   Na+ efflux (flow out of the cell) occurs.
D.   PNa+ becomes much greater than PK+.
E.   PK+ is the same as PNa+.
Question #30
The relative refractory period of an axon coincides with the period of
A.   increased K+ flux into the cell.
B.   activation and inactivation of voltage-dependent Na+ channels.
C.   Increased Na+ flux through K+ channels.
D.   increased K+ permeability of the cell.
E.   Na+ permeability that is greater than that during the depolarization phase.
Question #31
A postsynaptic neuron has three presynaptic inputs—from neurons X, Y, and Z. Stimulation of neuron X causes the postsynaptic neuron to depolarize by 0.5 mV. When X and Y are stimulated simultaneously, the postsynaptic neuron depolarizes by 1 mV. When X and Z are stimulated simultaneously, however, there is no change in the membrane potential of the postsynaptic neuron. What is most likely true about presynaptic neurons Y and Z?
A.   Y is excitatory and Z is inhibitory.
B.   They are both excitatory.
C.   They are both inhibitory.
D.   Z is excitatory and Y is inhibitory.
Question #32
Nicotinic receptors are expressed on skeletal muscle cells. Which of the following would likely be a side-effect of the use of nicotine, an agonist for those receptors?
A.   Drowsiness
B.   Muscle paralysis
C.   Muscle relaxation
D.   None of the answer choices are correct
E.   Muscle cell twitches (contractions)
Question #33
Which of the following statements regarding whole-muscle contraction is TRUE?
A.   The order of motor unit recruitment is independent of the size of the alpha motor neuron that innervates them.
B.   The order of recruitment of motor units in a muscle is such that the first units recruited generate the most tension.
C.   The order of recruitment of motor units in a muscle is such that the last units recruited are those that fatigue most readily.
D.   Motor units whose motor neurons have large-diameter cell bodies are recruited first, while motor units with smaller-diameter motor neurons are only activated as the level of activation in the spinal cord increases.
E.   Recruitment of one fast-glycolytic motor unit provides a smaller increment in whole-muscle tension than recruitment of one slow-oxidative motor unit.
Question #34
Sarah is a sprinter who specializes in quick and powerful bursts of speed followed by periods of rest. Priya is a marathon runner who specializes in long, steady runs. Compared to Priya, Sarah is likely to have
A.   lower concentrations of glycolytic enzymes in her leg muscles.
B.   hypertrophy of type I muscle fibers.
C.   a higher density of capillaries in her leg muscles.
D.   leg muscles with a larger diameter.
E.   leg muscles with a smaller diameter.
Question #35
During an isometric twitch in a skeletal muscle
A.   sarcomeres do not significantly shorten.
B.   the whole muscle shortens.
C.   tetanus occurs.
D.   H zones shorten.
E.   tension generated by the muscle always exceeds the load on the muscle.
Question #36
Binding of ________ to myosin permits cross-bridge ________ between actin and myosin in skeletal muscle cells.
A.   ATP; detachment
B.   calcium; detachment
C.   calcium; attachment
D.   ATP; attachment
E.   actin; detachment
Question #37
How is the strength of a stimulus encoded by neurons?
A.   By whether the action potential peak is positive or negative
B.   By the duration of action potentials
C.   By the size of action potentials
D.   By the frequency of action potentials
Question #38
What is the best description of a tetanic contraction in a skeletal muscle cell?
A.   Multiple action potentials in the motor neuron cause a sustained contraction.
B.   Repeated action potentials from the motor neuron summate into a sustained depolarization of the motor end plate, causing a sustained contraction.
C.   A single action potential in the motor neuron causes a sustained contraction.
D.   A very large amplitude action potential in the motor neuron causes a very strong contraction in the skeletal muscle cell.
E.   The action potential in the muscle cell is prolonged to last as long as the contraction.
Question #39
A scientist labels ACh with fluorescent dye to be able to visualize under the microscope where ACh travels and binds. She takes an image of the motor end plate and see that ACh is
A.   bound to receptors on the sarcoplasmic reticulum.
B.   bound to Ca2+ ions.
C.   bound to acetylcholinesterase in the end plate membrane.
D.   bound to muscarinic receptors at the end plate.
E.   inside the muscle cell sarcoplasm.
Question #40
How is the length of a skeletal muscle cell related to the force it can generate?
A.   The shorter a skeletal muscle cell is when it begins to contract, the stronger the force generation will be.
B.   The longer a skeletal muscle cell is when it begins to contract, the stronger the force generation will be.
C.   Skeletal muscle cells generate the most force when the contraction occurs at an intermediate length.
D.   Skeletal muscle cells generate the same amount of force, regardless of their length.
E.   The tension in a skeletal muscle cell is greatest when contractions occur at either very short or very long lengths.
Question #41
Which of the following statements regarding contraction in skeletal and smooth muscle is TRUE?
A.   In skeletal muscle, calcium initiates contraction by binding to myosin light-chain kinase, while in smooth muscle calcium initiates contraction by binding directly to tropomyosin.
B.   In skeletal muscle, calcium initiates contraction by binding to troponin, while in smooth muscle calcium initiates contraction by binding directly to myosin.
C.   In skeletal muscle, calcium ions bind to a regulatory protein on thin filaments; in smooth muscle, calcium ions bind to a regulatory protein on thick filaments.
D.   All of the choices are true.
E.   In skeletal muscle, calcium initiates contraction by binding to troponin, while in smooth muscle calcium initiates contraction by binding to calmodulin.
Question #42
Which best describes a role of L-type Ca2+ channels in cardiac muscle cells?
A.   They act as non-conducting voltage sensors that mediate excitation-contraction coupling.
B.   They are responsible for preventing tetanic contractions.
C.   They cause the absolute refractory period to be very brief.
D.   They are directly responsible for making cardiac muscle fatigue-resistant.
E.   They function exactly the same in cardiac muscle cells as they do in skeletal muscle.
Question #43
Which of the following statements about EPSPs is FALSE?
A.   They depolarize postsynaptic cell membranes.
B.   They are always the same amplitude.
C.   They are produced by the opening of ligand-gated sodium channels.
D.   They transmit signals over relatively short distances.
E.   They are able to summate.
Question #44
The chemical curare is a nicotinic receptor antagonist. How will curare affect skeletal muscle function?
A.   Muscle function is fine, but it will cause a loss of voluntary control.
B.   It activates an autoimmune disease that destroys myelin.
C.   It will cause persistent twitches with short periods of rest in between.
D.   It will cause flaccid paralysis (no muscle contraction possible).
E.   It will cause spastic paralysis (sustained, unwanted muscle contraction).

Need help with your exam preparation?